LSAT and Law School Admissions Forum

Get expert LSAT preparation and law school admissions advice from PowerScore Test Preparation.

General questions relating to LSAT Logical Reasoning.
 william92
  • Posts: 6
  • Joined: Mar 16, 2018
|
#44649
Hello,

I read the article regarding SA and NA which is below.
No one is admitted to Harvard without a top-1% score. Jane just took a PowerScore class, and everyone who takes a PowerScore class is guaranteed a 173. Therefore, Jane will be admitted to Harvard next year.

Conditionally, the argument can be diagrammed as follows:

Premise: Harvard → top-1% score

Premise: Jane → PowerScore → 173

Conclusion: Jane → Harvard
When I symbolize each element in the argument, It would be like,
P: A → B
P: C → D → E
C: C → A

and the article says the answer for SA is 173 → Harvard admission, which can be shown as E → A.
I do understand how this answer was produced but when I tried to simply (more like blindly) apply The Mechanistic Approach, the process left me elements B, D and E after ignoring common elements in both premises and conclusion (C and A) and eventually led me to the answer for NA; 173 → top-1% score (E → B). Is this answer for NA also a correct answer for Justify the Conclusion as sufficient assumption as there could be an overlap between SA and NA? (like, bare minimum for sufficient assumption to meet in order to make the conclusion valid but does not contain any extraneous info?) Would you help clarify this?
User avatar
 Jonathan Evans
PowerScore Staff
  • PowerScore Staff
  • Posts: 727
  • Joined: Jun 09, 2016
|
#44706
Hi, William,

Thanks for the question! (For reference, the entire article may be found at: https://blog.powerscore.com/lsat/bid/25 ... ifferences)

As the article indicates, the necessary assumption answer, 173 :arrow: top 1%, would not work for a Justify the Conclusion answer:
Assumption: 173 → top-1% score

This statement is not sufficient, of course, to prove the conclusion. Even if 173 were a top-1% score, that would only establish that Jane has satisfied a requirement necessary for admission into Harvard.
Yes, there can be situations in which a correct Justify the Conclusion answer (an assumption sufficient to justify a conclusion) could also be correct for an Assumption question (an assumption necessary for the conclusion to be valid).

For example:
  • Premise: IBM computers use Intel microprocessors.
    Conclusion: IBM computers are powerful.
    Assumption: Computers that use Intel microprocessors are powerful.
The assumption above is both necessary for the conclusion to be valid and sufficient to justify the conclusion.

Try out the Assumption Negation Test™: Computers that use Intel microprocessors are not powerful. Clearly, this makes the conclusion nonsensical and passes the test.

The statement also is sufficient to justify the conclusion. If we know that IBM computers use Intel microprocessors, then the fact that we also know computers that use Intel microprocessors are powerful is sufficient to establish that IBM computers are powerful.

The key is to focus on the task of the question at hand and not to confuse Assumption questions with Justify the Conclusion questions.

I hope this helps!
 william92
  • Posts: 6
  • Joined: Mar 16, 2018
|
#44778
Hello, Jonathan.

Appreciate your comments. Yes, I do understand how two different sufficient assumption and necessary assumption are produced. But, out of pure curiosity, how would you apply the mechanistic approach on the question in the article? Now I'm familiar with the mechanistic approach and I know it always produces correct answer for Justify questions but as I stated before, when I just simply apply it on the argument in the article, it leads me to the necessary assumption. I'll wait for the response. Thanks!
 Adam Tyson
PowerScore Staff
  • PowerScore Staff
  • Posts: 5387
  • Joined: Apr 14, 2011
|
#44786
Applying the mechanistic approach to this question could cause some confusion, william92! Typically we talk about that approach as being a way of connecting "rogue" or "new" elements from the premises to rogue or new elements in the conclusion. Let's see what that looks like here:

The premises talk about Harvard, top 1%, Jane, a PowerScore class, and a 173 on the LSAT.

The conclusion talks about Jane and Harvard.

Uh-oh - there is nothing new in the conclusion! Jane and Harvard were both mentioned in the premises, so those are not new or rogue elements. Can we use the mechanistic approach at all, or must we find another way?

Yes, we can use it, and yes, we use it by finding another way!

Let's look at it through a different lens. We want to prove, or justify, the claim that Jane will get into Harvard. Harvard, therefore, must be a necessary condition in our prephrase and in the correct answer choice. That premise about the top 1% has Harvard as a SUFFICIENT condition, so that isn't helping us any. Harvard might guarantee a lot of things, but none of that will help us to find out what it is that guarantees Harvard. Since there is no way that this premise can help us, let's just ignore that premise. Now what do we have?

The premises speak of Jane, a PowerScore class, and a 173 on the LSAT.

The conclusion speaks of Jane and of Harvard.

Mechanistically, I now want to connect either the PowerScore class, or the 173, or both, to Harvard. Jane can be ignored, since she is in the premises and in the conclusion. My prephrase must be "I need that class and/or that score to be sufficient for admission to Harvard."

As you can see, the mechanistic approach will work, but only after we jettison the distraction created by the first premise. If I wanted to prove something about getting a top 1% score, I would need that premise, but since my conclusion has nothing to do with that element, it's just there to cause me trouble.

Knowing what to focus on and what to reject or ignore will take practice, but you'll get there if you keep at it. Good luck!
 william92
  • Posts: 6
  • Joined: Mar 16, 2018
|
#44807
Thanks for the clarification Adam!

Yes, I could indeed produced the correct sufficient assumption when I think of the first premise as a distractor element which is not essential to prove the conclusion. It helps a lot!

Get the most out of your LSAT Prep Plus subscription.

Analyze and track your performance with our Testing and Analytics Package.